Discuss events which are simultaneous in one frame?

In summary, the idea of simultaneity is often discussed in relation to the theory of relativity, specifically in the context of how events can be perceived differently by observers in different frames of reference. There are two main types of simultaneity - reception simultaneity, where two events are perceived as happening at the same time by an observer, and transmission simultaneity, where the events were actually released at the same time but may not be perceived as simultaneous due to the time it takes for light to travel. The concept of simultaneity is important in understanding how time is perceived in different frames of reference, and it is often discussed in relation to the Lorentz invariance of the fundamental laws of physics.
  • #141
Neopolitan:
Note that this is all speculation. I am aware that absolutes are not part of relativity and I am not saying there are any. I just would like to hear why there can't be any.
Formally in relativity, nowhere does it say there is no absolute frame. What is said is that 'it is impossible to detect inertial motion without reference to another frame'.
In other words, all motion is relative. Therefore, in practice, even if an 'absolute frame' existed, it would not make any difference to the way we percieve things.

(I'm putting on my tin-hat and waiting for the onslaught of the enraged relativists).
 
Physics news on Phys.org
  • #142
neopolitan said:
Given that I know the relative velocities of NIM1 and NIM2, will I be able to use their observations to accurately locate Event F (in my reference frame), if I happened to not notice it myself? I can work out when and where each actually is (in my reference frame) when they observe Event F and I know how their perceptions of time and space are skewed by their relative motions.
Certainly. In fact, you only need one NIM observer. Simply Lorentz transform his coordinates into yours.

neopolitan said:
What, if anything, makes this "one true rest frame" totally impossible? As far as I can tell, you could never detect this "one true rest frame" - or absolute at rest (AAR) frame - since the skewing of all other frames, each of which is nominally at rest in terms of itself, will make the AAR frame appear like any other NIM frame. However, this does not make the AAR frame impossible per se. Does it?

Both of them may be in motion relative to some indetectible AAR frame, making both of them absolutely in motion (AIM) relative to the AAR frame. This does not stop us from nominating one of them as "at rest" and the other as "in motion". Despite this arbitrary assignment of NAR and NIM frames, would we not still work out the "absolute" space-time locations of events? - we would just express them in terms of our own frames. Chronos and we would agree where all events take place, we would just express those events differently.
This is essentially the Lorentz ether theory, which is experimentally indistinguishable from special relativity. Analytically it goes something like this: set up all reference frames, randomly pick any one, call it the "ether frame", proceed from that frame to make all special relativity predictions. I have no problem with people who prefer the Lorentz interpretation to the Einstein interpretation. Personally, I think it is wise to learn both and use whichever fits the situation best. In my case, I like the Lorentz interpretation for explaining relativistic Doppler effects, but the Einstein interpretation is for most other things.
 
  • #143
DaleSpam said:
Certainly. In fact, you only need one NIM observer. Simply Lorentz transform his coordinates into yours.

Indeed, I explained why I had three later in the post.

DaleSpam said:
This is essentially the Lorentz ether theory, which is experimentally indistinguishable from special relativity. Analytically it goes something like this: set up all reference frames, randomly pick any one, call it the "ether frame", proceed from that frame to make all special relativity predictions. I have no problem with people who prefer the Lorentz interpretation to the Einstein interpretation. Personally, I think it is wise to learn both and use whichever fits the situation best. In my case, I like the Lorentz interpretation for explaining relativistic Doppler effects, but the Einstein interpretation is for most other things.

This is not quite what I am asking. I know that you could select any inertial frame and use that as what could be called an "ether frame". But what I am asking is, is there anything preventing a "one true rest frame" or an "absolute (at) rest frame"?

Then going further, if there is nothing preventing such a frame (even if we cannot distinguish it), would not that frame's hypersurface of simultaneity be the boundary of the universe? It seems to me that the boundary of the universe is more of a "when" question than a "where" question.

This would mean that, in the terms that we normally use for thinking about such things, there would be no 3 dimensional edge to the universe and no 3 dimensional centre. Instead there would be a 4 dimensional edge and a 4 dimensional centre. (It may help to remove one dimension and think of a sphere. The two dimensional surface of the sphere has no centre and no edge. The three dimensional sphere itself, however, has the two dimensional surface as the boundary and the centre of the sphere is surrounded by and separated from the surface - so two dimensional people living on the sphere would never be able to reach the centre of the universe. The centre of our universe would, therefore, be in the past - in a big bang event, or some equivalent to a big bang event.)

The standard cosmological model has the universe expanding like the surface of a balloon. Is what I have expressed above just saying the same thing, perhaps in another way?

I wonder if a few people thought I was wandering away from the simultaneity topic. I wasn't after all. :smile:

cheers,

neopolitan
 
  • #144
neopolitan said:
Then going further, if there is nothing preventing such a frame (even if we cannot distinguish it), would not that frame's hypersurface of simultaneity be the boundary of the universe?
You have to be very careful here. My GR is not strong enough to provide a lot of detail or arguments, but everything I have been describing here has been exclusively SR. If you are going to be talking about the universe as a whole then you cannot simply use SR and you would need to use GR.

My understanding is that the notion of simultaneity is much more difficult to define in GR than in SR. In other words, there are many spacetimes that you simply cannot draw a continuous surface of simultaneity that covers the whole spacetime. For example, in a rotating spacetime a surface of simultaneity unavoidably has a discontinuous jump along some radius.

However, even if the universe has a simple geometry for which a universal notion of simultaneity could be defined, your "true boundary" of the universe and "true center" of the universe would have no physical significance. Any arbitrary reference frame could construct its "center" and "boundary" which would be equally valid and experimentally indistinguishable.
 
Last edited:
  • #145
I've never seen a convincing argument that simultaneity has any physical significance whatsoever.

Events with the same time coordinate in a given coordinate system can be regarded as simultaneous, but the choice of coordinate systems is a purely arbitrary human choice.

To me this implies that simultaniety is also a human choice, a matter of labels on a map, rather than having any fundamental significance.
 
  • #146
pervect said:
I've never seen a convincing argument that simultaneity has any physical significance whatsoever.

Events with the same time coordinate in a given coordinate system can be regarded as simultaneous, but the choice of coordinate systems is a purely arbitrary human choice.

To me this implies that simultaniety is also a human choice, a matter of labels on a map, rather than having any fundamental significance.
I agree completely. The universe simply doesn't care about simultaneity as far as I can tell.
 
  • #147
I have read this a few times, that "simultaneity has no physical significance".

I wasn't necessarily saying that simultaneity does have "physical significance" (although I am not 100% what you mean by that, so I can't say whether I agree or disagree). If you could please look at what I did say, can you then make comment on whether the boundary of the universe has any physical significance?

DaleSpam said:
My understanding is that the notion of simultaneity is much more difficult to define in GR than in SR. In other words, there are many spacetimes that you simply cannot draw a continuous surface of simultaneity that covers the whole spacetime. For example, in a rotating spacetime a surface of simultaneity unavoidably has a discontinuous jump along some radius.

I note an earlier comment DaleSpam made in the post quoted, about insufficiently strong GR knowledge, so this is not an attack - I just want to work through something here.

What exactly is "a rotating spacetime"? What is meant by "many spacetimes"? Is "a spacetime" used here to replace "an inertial frame" (noting that frames in GR don't have to be inertial)?

My understanding, which may be wrong, is that the universe doesn't care what you do in it (as equally as it doesn't care about simultaneity). I can choose whatever frame of reference I like, I can consider ourselves to be at rest, and I can subsequently work out the spacetime locations of events around me, in terms of my selected frame of reference. Other observers can follow the same process, choosing whatever frame of reference they like, considering themselves to be at rest and they can subsequently work out the spacetime locations of events around them, in terms of their own selected frames of reference.

If we all then consider the same event, we can use transformations between our frames of reference and work out that we are indeed considering the same event. Is this correct?

If it is correct, then I don't see what relevance there is to "a rotating spacetime". I don't see how it relates to what I was initially pondering, the possibility of a "one true rest frame" (or "absolute at rest frame", or AAR frame) which is indistinguishable from any "nominally in motion" NIM frame. I don't think there is any reason to assume that an AAR frame would be, or should be, rotating. It could be, I guess. You are just left with the question "what is it rotating in reference to?" Since this is a conceptual "absolute at rest" frame, it is at rest. So really what you would be saying is that all other frames, which appear to be inertial, are for some reason actually rotating relative to the AAR frame (actually, they would be orbiting - the vast majority of them impossibly, since their rotation velocity would be greater than the speed of light).

cheers,

neopolitan
 
Last edited:
  • #148
Any real GR expert reading this, please feel free to correct any mistakes I make. This is all according to my rather uninformed understanding.
neopolitan said:
What exactly is "a rotating spacetime"? What is meant by "many spacetimes"? Is "a spacetime" used here to replace "an inertial frame" (noting that frames in GR don't have to be inertial)?

My understanding, which may be wrong, is that the universe doesn't care what you do in it (as equally as it doesn't care about simultaneity). I can choose whatever frame of reference I like, I can consider ourselves to be at rest, and I can subsequently work out the spacetime locations of events around me, in terms of my selected frame of reference. Other observers can follow the same process, choosing whatever frame of reference they like, considering themselves to be at rest and they can subsequently work out the spacetime locations of events around them, in terms of their own selected frames of reference.
In GR it is a little more complicated than that. Sure, you can draw whatever coordinate system you want, but the underlying spacetime itself is curved. This means that certain complexities will arise in any coordinate system that you use.

Let's consider 26 inertial clocks, A-Z, arranged in a stable ring in flat spacetime (SR). Clock A synchronizes with clock B using Einstein synchronization, then B with C, ... When we get around the ring we find, as expected, Z is synchronized with A. In flat spacetime there is a well-defined global notion of simultaneity.

Consider the same inertial clocks in stable orbit around a non-rotating massive body (curved spacetime) and further consider that they are close enough to each other to consider the spacetime between any two neighbors to be flat. A synchronizes with B which synchronizes with C ... When we get around the ring we find again that Z is synchronized with A. Again, there is a well-defined global notion of simultaneity.

Now, consider the same inertial clocks in stable orbit around a rotating massive body (rotating spacetime). A synchronizes with B ... When we get around the ring, due to the frame dragging effect, we find that Z is not synchronized with A. There is no well-defined global notion of simultaneity, simultaneity can only be defined locally in a rotating spacetime.
 
Last edited:
  • #149
Hi DaleSpam,

DaleSpam said:
Any real GR expert reading this, please feel free to correct any mistakes I make. This is all according to my rather uninformed understanding.

neopolitan said:
What exactly is "a rotating spacetime"? What is meant by "many spacetimes"? Is "a spacetime" used here to replace "an inertial frame" (noting that frames in GR don't have to be inertial)?

My understanding, which may be wrong, is that the universe doesn't care what you do in it (as equally as it doesn't care about simultaneity). I can choose whatever frame of reference I like, I can consider ourselves to be at rest, and I can subsequently work out the spacetime locations of events around me, in terms of my selected frame of reference. Other observers can follow the same process, choosing whatever frame of reference they like, considering themselves to be at rest and they can subsequently work out the spacetime locations of events around them, in terms of their own selected frames of reference.

In GR it is a little more complicated than that. Sure, you can draw whatever coordinate system you want, but the underlying spacetime itself is curved. This means that certain complexities will arise in any coordinate system that you use.

Let's consider 26 inertial clocks, A-Z, arranged in a stable ring in flat spacetime (SR). Clock A synchronizes with clock B using Einstein synchronization, then B with C, ... When we get around the ring we find, as expected, Z is synchronized with A. In flat spacetime there is a well-defined global notion of simultaneity.

Consider the same inertial clocks in stable orbit around a non-rotating massive body (curved spacetime) and further consider that they are close enough to each other to consider the spacetime between any two neighbors to be flat. A synchronizes with B which synchronizes with C ... When we get around the ring we find again that Z is synchronized with A. Again, there is a well-defined global notion of simultaneity.

Now, consider the same inertial clocks in stable orbit around a rotating massive body (rotating spacetime). A synchronizes with B ... When we get around the ring, due to the frame dragging effect, we find that Z is not synchronized with A. There is no well-defined global notion of simultaneity, simultaneity can only be defined locally in a rotating spacetime.

None of this answers my questions. I understand what you mean by "rotating spacetime" and I accept that it may only be possible to define simultaneity "locally in a rotating spacetime" but I fail to see the relevance.

If anything, it gives me reason to wonder if the idea of an AAR has more relevance than I initially though, since your explanation forces me to ask this:

Accepting what you have to say about "rotating spacetimes", in reference to what is this spacetime rotating?

I know that GR is not where you are at at the moment, but it seems that you didn't address the second paragraph of mine that you quoted. Can you or can you not, given details of another frame's relative motion (inertial, "rectilinear and non-rotating" or curved/rotating), work out where events are in your own frame of reference and confirm that other frame's interpretation of events are valid? Or are you suggesting that, if you are in a rotating spacetime, you cannot perform a transformation to obtain that event's spacetime location in terms of an observer in an inertial, rectilinear and non-rotating frame?

Note, this is pretty much the same question as I asked before, viz (with slight editing)

If we (all observers in our individual frames) all ... consider the same event, we can use transformations between our frames of reference and work out that we are indeed considering the same event. Is this correct?

cheers,

neopolitan
 
  • #150
Forgive me chipping in here, but you've gone a bit off track from simultaneity.
What exactly is "a rotating spacetime"? What is meant by "many spacetimes"? Is "a spacetime" used here to replace "an inertial frame" (noting that frames in GR don't have to be inertial)?
In GR, we refer to any particular metric as a 'space-time'. A metric can define an entire universe. An observer is someone inhabiting the space-time, who uses a local ( usually Minkowski) co-ordinate system to relate to things close to her.

My understanding, which may be wrong, is that the universe doesn't care what you do in it (as equally as it doesn't care about simultaneity). I can choose whatever frame of reference I like, I can consider ourselves to be at rest, and I can subsequently work out the spacetime locations of events around me, in terms of my selected frame of reference. Other observers can follow the same process, choosing whatever frame of reference they like, considering themselves to be at rest and they can subsequently work out the spacetime locations of events around them, in terms of their own selected frames of reference.
That's what happens in practice. We each have a laboratory frame for close-in work, but in astronomy we might choose a frame with the sun at the center.

If we all then consider the same event, we can use transformations between our frames of reference and work out that we are indeed considering the same event. Is this correct?
Yes.

I think you're making heavy weather of this simultaneity thing. What does it matter if two observers disagree about two events being simultaneous or not ?
 
  • #151
Mentz114 said:
Forgive me chipping in here, but you've gone a bit off track from simultaneity.

<snip>

I think you're making heavy weather of this simultaneity thing. What does it matter if two observers disagree about two events being simultaneous or not ?

I refer you to post #143. Hopefully that will make clear what mattered and you can see that I personally wasn't wandering away from simultaneity.

Here are the relevant paragraphs, but the whole lot in glorious context lies below.

neopolitan said:
I know that you could select any inertial frame and use that as what could be called an "ether frame". But what I am asking is, is there anything preventing a "one true rest frame" or an "absolute (at) rest frame"?

Then going further, if there is nothing preventing such a frame (even if we cannot distinguish it), would not that frame's hypersurface of simultaneity be the boundary of the universe? It seems to me that the boundary of the universe is more of a "when" question than a "where" question.

This has not been addressed.

I agree that the discussion about spacetime is a little off the track but in post #147 I did again try to bring it back -

neopolitan said:
My understanding, which may be wrong, is that the universe doesn't care what you do in it (as equally as it doesn't care about simultaneity). I can choose whatever frame of reference I like, I can consider ourselves to be at rest, and I can subsequently work out the spacetime locations of events around me, in terms of my selected frame of reference. Other observers can follow the same process, choosing whatever frame of reference they like, considering themselves to be at rest and they can subsequently work out the spacetime locations of events around them, in terms of their own selected frames of reference.

If we all then consider the same event, we can use transformations between our frames of reference and work out that we are indeed considering the same event. Is this correct?

If it is correct, then I don't see what relevance there is to "a rotating spacetime". I don't see how it relates to what I was initially pondering, the possibility of a "one true rest frame" (or "absolute at rest frame", or AAR frame) which is indistinguishable from any "nominally in motion" NIM frame. I don't think there is any reason to assume that an AAR frame would be, or should be, rotating. It could be, I guess. You are just left with the question "what is it rotating in reference to?" Since this is a conceptual "absolute at rest" frame, it is at rest. So really what you would be saying is that all other frames, which appear to be inertial, are for some reason actually rotating relative to the AAR frame (actually, they would be orbiting - the vast majority of them impossibly, since their rotation velocity would be greater than the speed of light).

You effectively answered the question in the second paragraph, thanks. Now if I could get someone to address the other questions in #143 without spiralling off into what is really another topic, it would great! :smile:

cheers,

neopolitan
 
  • #152
But what I am asking is, is there anything preventing a "one true rest frame" or an "absolute (at) rest frame"?
Hmm. A frame of reference is not a physical thing. It has to be associated with some observer who is using coordinates of her choice. I'm not sure what an absolute frame could mean. If the universe was filled with some kind of fluid that was flow free, one could use it a frame of reference, and we could define absolute velocity. But the universe is not filled in such a way. So probably the notion of an absolute frame is not useful.

Then going further, if there is nothing preventing such a frame (even if we cannot distinguish it), would not that frame's hypersurface of simultaneity be the boundary of the universe? It seems to me that the boundary of the universe is more of a "when" question than a "where" question.
The entirety of the universe has no boundary by definition. What can be outside the universe to make a boundary ? But your question makes sense if we consider the limits of our observable universe. You are aware that the further light travels to us the more red-shifted it is. So there is a distance from us at which this light becomes undetectable, and this is a boundary in the sense that we cannot see past it. Because the boundary is defined in terms of light propagation, the distance is also a time.
If we pointed our two telescopes in opposite directions and received a signal with identical red-shift ( say an H2 Lyman line) in both telescopes at the same time on our clock, I'm not sure if we could say that the light had been emitted 'simultaneously'. It doesn't have much physical import.

I may have repeated stuff from earlier posts, I've read through but I don't remember it all.
 
Last edited:
  • #153
Mentz114 said:
But what I am asking is, is there anything preventing a "one true rest frame" or an "absolute (at) rest frame"?

You don't need to make an absolute frame out of empty space. There need to be landmarks before we can have a preferred frame, there needs to be a mass in it. You can make flat space into a fixed frame, but there is no need for it. Not in GR. You need to use frames only in curved space.
 
  • #154
neopolitan said:
Accepting what you have to say about "rotating spacetimes", in reference to what is this spacetime rotating?
Rotation is intrinsic. You don't have to be rotating wrt anything to be rotating. If you are in a closed laboratory you can do all sorts of experiments to determine if you are rotating or not. You don't need to refer to anything outside the laboratory.

Don't get too sidetracked about the details of a rotating spacetime. The point is simply that in GR there is no concept of simultaneity that makes sense universally. I think it is problematic at best to talk about "boundaries" and "center" of the universe in terms of a concept that doesn't apply universally.
 
Last edited:
  • #155
neopolitan said:
None of this answers my questions. I understand what you mean by "rotating spacetime" and I accept that it may only be possible to define simultaneity "locally in a rotating spacetime" but I fail to see the relevance.

If anything, it gives me reason to wonder if the idea of an AAR has more relevance than I initially though, since your explanation forces me to ask this:

Accepting what you have to say about "rotating spacetimes", in reference to what is this spacetime rotating?
Are you sure you're familiar with the notion of "rotating spacetime"? Note that it has nothing to do with what coordinate system you choose, it is a physical feature of the spacetime itself. The rotating universe is a GR solution that was originally discovered by Kurt Godel (of Godel's incompleteness theorem in logic) to allow "closed timelike curves", i.e. time travel into the past; I think this may be the key reason it is not possible to come up with a global definition of simultaneity in such a spacetime. Physicists refer to the idea of dividing up a 4D spacetime into a stack of 3D spacelike hypersurfaces as a "foliation" of that spacetime, and if I'm remembering correctly I think it's only possible to foliate a spacetime which is globally hyperbolic (and the third paragraph here seems to confirm my memory), with part of the definition of globally hyperbolic spacetimes being that they do not contain closed timelike curves.

Here is a nice essay on the Godel rotating universe solution, written by someone who wrote a Ph.D. thesis on the subject. The author gives a good explanation of what it means physically for the universe as a whole to be rotating, and why it does not require a center of rotation:
When I tell people about the possibility of a rotating universe, their reaction is usually either a silly smile, or the very well motivated question: With respect to what would the universe rotate? I viciously reply: With respect to something that does not rotate, that is, something that does not experience any centrifugal forces. OK, this is correct, but it needs some elaboration.

First of all, don't try to imagine the universe as rotating as a whole. That way of thinking is misleading. I'll come back to rotation as a whole later.

Second, don't think that this implies some center of rotation. According to the Copernican principle, all places in the universe are equivalent. This is a simplifying assumption adopted by most cosmologists; whether it holds in reality is an open question. On smaller scale the universe is badly inhomogeneous, but there is still hope that the large scale structure is homogeneous.

Third, study carefully the following attempt to visualize a rotating universe.

Imagine you are in a laboratory without windows floating around somewhere in the universe. If you and the other objects in the laboratory get pressed against the walls, you would say that the laboratory is rotating, and centrifugal forces are responsible for the effects. Now, the laboratory happens to be equipped with small engines that can be used to control the rotation. Use the engines until you have totally eliminated the centrifugal forces, and thereby the rotation. When done, drill some peepholes in the laboratory (but please make sure you don't lose your air supply). Observe the galaxies. If you find that the galaxies rotate around you, then the universe is said to be rotating.

You have actually only seen that the universe rotates around the point where you are, but if the Copernican principle holds, then it rotates around any point. That's a rotating universe.

So keep in mind that when I talk about a rotating universe, I mean that the matter of the universe rotates around the non-rotating observer. There is a better word for it: vorticity. In classical hydrodynamics, the vorticity w of a velocity field v is defined using the rot operator:

click for image of equation

In general relativity, there is a similar definition. One expresses the vorticity four-vector in terms of matter four-velocity field (a four-vector is a vector with one 'time' and three 'space' components).
 
Last edited:
  • #156
neopolitan said:
This is not quite what I am asking. I know that you could select any inertial frame and use that as what could be called an "ether frame". But what I am asking is, is there anything preventing a "one true rest frame" or an "absolute (at) rest frame"?
As a metaphysical belief this is possible, but it's meaningless as a physical theory. You're free to imagine that one coordinate system is "metaphysically preferred" in the sense that its judgments about simultaneity (or about other frame-dependent questions, like which of two objects has a higher speed) represent the "real truth" of the matter. But relativity says that no coordinate system is physically preferred over any other, so you can never have any empirical evidence to justify the idea that one frame's judgments are better than any other's. And as far as metaphysics goes, if you like the principle of Occam's razor than the fact that a metaphysically preferred frame would have no empirical consequences whatsoever would be a pretty good argument for dispensing with such a notion, although nothing forces you to accept the Occam's razor argument.
neopolitan said:
This would mean that, in the terms that we normally use for thinking about such things, there would be no 3 dimensional edge to the universe and no 3 dimensional centre. Instead there would be a 4 dimensional edge and a 4 dimensional centre. (It may help to remove one dimension and think of a sphere. The two dimensional surface of the sphere has no centre and no edge. The three dimensional sphere itself, however, has the two dimensional surface as the boundary and the centre of the sphere is surrounded by and separated from the surface - so two dimensional people living on the sphere would never be able to reach the centre of the universe. The centre of our universe would, therefore, be in the past - in a big bang event, or some equivalent to a big bang event.)

The standard cosmological model has the universe expanding like the surface of a balloon. Is what I have expressed above just saying the same thing, perhaps in another way?
The standard cosmological model actually allows for three possible "shapes" of an expanding universe, depending on the density of matter and energy throughout space. Also, when you talk about a "4 dimensional center", it seems like you're imagining the universe as the 3D surface of a 4D hypersphere which is sitting in a larger 4D space--this is what would be called an "embedding space", but the mathematics of differential geometry actually allows you to describe the curvature of a 3D surface without the need for it to be curved in a higher-dimensional space. I discussed both these points in post #4 here:
According to the Big Bang theory, the Big Bang was not an explosion in a preexisting 3-dimensional space, with matter and light expanding out into empty space from some central point--instead, matter and energy are understood to fill all of 3D space, and what's expanding is space itself. The key is to understand that the Big Bang theory is based on Einstein's theory of general relativity, which explains gravity in terms of matter/energy causing spacetime to become curved--depending on the average density of matter/energy throughout the universe, a consequence of this is that the universe as a whole can be curved, with either positive curvature, zero curvature, or negative curvature. For a closed universe with positive curvature, you can visualize it if you drop the dimensions by one--instead of curved 3-dimensional space, which is impossible for us to visualize, picture a 2D universe a la Flatland in which 2D space is actually curved into a sphere, and "expanding space" means that the sphere is blowing up like a balloon while the bits of 2D matter on the surface do not change in size. You can see that if you pasted a bunch of bits of paper on a balloon and then blew it up, each bit would see the other bits receding from it, just like what we see with other galaxies. If you play the movie backwards so that the size of the sphere approaches zero, you can seen that all the bits of matter throughout the universe get more and more squished together, approaching infinite density as the size approaches zero--this is what the big bang is supposed to be. Of course, this analogy forces you to picture the 2-dimensional surface of the sphere expanding in a higher 3rd dimension, and while it is possible that our curved 3D space is expanding in some kind of higher 4D space, mathematically there is no need for such a thing--instead of describing the curvature of a surface with reference to a higher-dimensional "embedding space", it is possible to describe curvature using purely intrinsic features that could be observed by a being confined to the surface (like whether the sum of angles of a triangle drawn on the surface is more, less, or equal to 180 degrees), and general relativity uses only such intrinsic features to describe what it means for space to be curved (see this page on differential geometry, the mathematical basis for general relativity, which talks about the difference between intrinsic and extrinsic descriptions of curvature).

For a universe with zero curvature, picture an infinite chessboard in which all the squares are growing at the same rate, while the pieces at the center of each square remain unchanged in size. If you play the movie backwards, the distance between any two squares approaches zero as you approach the moment of the big bang, which means the density of the matter on the squares (represented by the chess pieces) approaches infinity as it gets smushed together more and more tightly. A universe with negative curvature would be something like an infinite saddle-shape which is a little harder to picture expanding, but if you can picture the other two you get the basic idea. From Ned Wright's Cosmology Tutorial, a graphic showing the 2D analogues of the three types of spatial curvature, negative, zero, and positive:

3geomtrs.gif
One other thing to point out is that even if you want to embed curved 3D space in a higher-dimensional euclidean space, or curved 4D spacetime in a higher-dimensional flat spacetime, one additional dimension may not be enough (as an analogy you might think of a 1D line curved into a corkscrew shape, which can't be embedded in 2D space). As discussed in this thread, it has been proven that any curved 4D spacetime could be embedded in a flat spacetime with 90 dimensions, 87 spacelike and 3 timelike. I don't know if anyone has come up with an example of a spacetime that would require this many dimensions to embed, but this is the upper bound.
 
  • #157
cyberdyno said:
You don't need to make an absolute frame out of empty space. There need to be landmarks before we can have a preferred frame, there needs to be a mass in it. You can make flat space into a fixed frame, but there is no need for it. Not in GR. You need to use frames only in curved space.

Cyberdyno - please correct post #153. I did not ask the question you attribute to me.

You can make flat space into a fixed frame, but there is no need for it.
Wrong. You just said that matter is required. Matter is always required to define a frame.
 
  • #158
Mentz114 said:
The entirety of the universe has no boundary by definition. What can be outside the universe to make a boundary ? But your question makes sense if we consider the limits of our observable universe. You are aware that the further light travels to us the more red-shifted it is. So there is a distance from us at which this light becomes undetectable, and this is a boundary in the sense that we cannot see past it. Because the boundary is defined in terms of light propagation, the distance is also a time.
If we pointed our two telescopes in opposite directions and received a signal with identical red-shift ( say an H2 Lyman line) in both telescopes at the same time on our clock, I'm not sure if we could say that the light had been emitted 'simultaneously'. It doesn't have much physical import.

Extremely red-shifted light will still be detectable. It's called infrared radiation. IR radiation is still the same sort of thing as light, it's just a "colour" that we can't see. What we normally refer to as light is just the spectrum of EMR that we can see unaided but we can use different telescopes and "see" any frequency in the EMR spectrum we want to.

I will get to the boundary issue in a moment.

JesseM said:
The standard cosmological model actually allows for three possible "shapes" of an expanding universe, depending on the density of matter and energy throughout space. Also, when you talk about a "4 dimensional center", it seems like you're imagining the universe as the 3D surface of a 4D hypersphere which is sitting in a larger 4D space--this is what would be called an "embedding space", but the mathematics of differential geometry actually allows you to describe the curvature of a 3D surface without the need for it to be curved in a higher-dimensional space. I discussed both these points in post #4 here:

One other thing to point out is that even if you want to embed curved 3D space in a higher-dimensional euclidean space, or curved 4D spacetime in a higher-dimensional flat spacetime, one additional dimension may not be enough (as an analogy you might think of a 1D line curved into a corkscrew shape, which can't be embedded in 2D space). As discussed in this thread, it has been proven that any curved 4D spacetime could be embedded in a flat spacetime with 90 dimensions, 87 spacelike and 3 timelike. I don't know if anyone has come up with an example of a spacetime that would require this many dimensions to embed, but this is the upper bound.

I actually have thought that one extra dimension is not sufficient. It has been a concern, since it seemed to lead to an infinite progression of dimensions.

It is interesting to see that 90 dimensions appears to be an upper limit. Personally, I cannot fathom why, once you get up to 90 dimensions, you should suddenly stop there. What is special about 90? If it were a more natural number I might feel slightly more comfortable about it: say 81, as either three to the power of six or 9 squared, or 128, as two to the power of seven, or 91 as the factorial of thirteen, or 100, as 10 squared, or 89, either as the nearest prime number or as the 12th number in the fibonacci sequence, or 85 as the seventh in a sequence of summed squares. It's difficult to see any significance to 90.

As for boundaries, my perception is that a 90-dimensional universe is unbounded (in terms of 90 dimensions). In terms of 89 dimensions, however, the universe would be bounded and so on, all the way down to the "useful" dimensions, if I can call them that.

(Note, I do see that this is an 89+1 dimensional universe, which would make more sense if there were 89 spacelike dimensions and 1 timelike. But you specified that it was 87 spacelike dimensions and 3 timelike. Finding something that makes sense of 87 and three is equally difficult. (29+1)*3? But then we are looking for a meaning for 29 and 3. I can live with 3 since we perceive our universe to have three spacelike dimensions. 29 as the tenth prime number? Ok, so we are left looking for a meaning for 10. The factorial of four? Why four and why a factorial, and why a prime number before? Anyway, I think you see the point, where is the physical significance of 90 dimensions?)

I agree with Mentz in so much as a four dimensional universe has no meaningful boundary in terms of four dimensions. However, a three dimensional universe does have a meaningful boundary in terms of four dimensions.

If you think about the surface of sphere, it is not bounded in two dimensions (although that two dimensional space is actually curved in terms of three dimensions). It is, however, "limited" or bounded in terms of three dimensions, when the radius of the sphere becomes apparent.

Similarly, the hypersurface of a hypersphere is not bounded in three dimensions (although that three dimensional space is actually curved in terms of four dimensions). Similarly, the hypersurface of the hypersphere is bounded in terms of four dimensions, when the "hyper-radius" of the hypersphere becomes apparent.

While mathematically you might be able to express this in terms where there is no need for a hypersphere or any other 4-D shape, but I am trying to interpret this in useful terms. So, when we don't observe any open surfaces in our perceived 3-D universe, other than in the world of mathematics, is there any reason to assume an open hypersurface in what we know to be (at the very least) a 4-D universe?

cheers,

neopolitan
 
  • #159
Extremely red-shifted light will still be detectable. It's called infrared radiation. IR radiation is still the same sort of thing as light, it's just a "colour" that we can't see. What we normally refer to as light is just the spectrum of EMR that we can see unaided but we can use different telescopes and "see" any frequency in the EMR spectrum we want to.
Are you saying there is no limit below which we cannot detect light ? That's just plain wrong. Our instruments are not infinitly sensitive. Already we have to cool the detectors to very low temperatures.

This thead has got ridiculous. From the relativity of simultaneity, which is simple and easy to grasp, we now have multi-dimensional cosmologies and all sorts of weird stuff that has nothing to do with the thread topic.

I'm unsubscribing from this farago.
 
  • #160
Mentz114 said:
Are you saying there is no limit below which we cannot detect light ? That's just plain wrong. Our instruments are not infinitly sensitive. Already we have to cool the detectors to very low temperatures.

It is largely irrelevant, but I think it is also largely wrong. I am pretty damn sure that we can detect all frequencies below the light spectrum at least down to the ELF radio spectrum - we might have problems with weak signals but not lowish frequencies. Admittedly you need a landmass as the detector (like a peninsular or a subcontinent), but it is technically feasible to detect an ELF signal.

Problems with detecting frequencies below ELF (below 1 hertz for example) would have nothing to do with the temperature of the detectors and more to do with the size of the detectors.

I don't think there is any transmitter in the universe that is moving fast enough to cause doppler shift down to below ELF, is there?

Note, I know this is entirely off topic. I will start a new thread on it - please respond to it there rather than here.

neopolitan said:
I don't know how to make a proper link! :(

But just above this is the link to the new thread

thanks,

neopolitan
 
Last edited:
  • #161
Neopolitan, I have to agree with Mentz, this tangent of yours is rather absurd.

First, as I already pointed out, there is no universal concept of synchronization with which to easily extract your 3D universal hypersurface. Synchronization is only generally clearly defined locally over small regions where spacetime is essentially flat.

Second, you are rather ignorant about the concept of embedding (not a criticism, I am too) so it is unwise to blindly assume that it is possible to use a fourth timelike dimension to embed even a simple 3D spacelike geometry and further unwise to claim that the result of such an embedding would be that "a three dimensional universe does have a meaningful boundary in terms of four dimensions". If you really wish to pursue this line of thought I would highly recommend that you study the embedding concept for a while and then actually do the math.

And third, so what? If we were 2D beings living on the surface of a 3D sphere what benefit would we get from projecting our space up into 3 dimensions. We would find that the "boundary" of our universe is everywhere and that the "center" of our universe is nowhere and that our universe is curved at a certain curvature. All of which we could have deduced intrinsically. I don't see the value added by the embedding.

PS Light can be red-shifted below zero frequency. That is what an event horizon is.
 
  • #162
DaleSpam said:
And third, so what? If we were 2D beings living on the surface of a 3D sphere what benefit would we get from projecting our space up into 3 dimensions. We would find that the "boundary" of our universe is everywhere and that the "center" of our universe is nowhere and that our universe is curved at a certain curvature. All of which we could have deduced intrinsically. I don't see the value added by the embedding.

Is it generally agreed that 'the "boundary" of our universe is everywhere and that the "center" of our universe is nowhere and that our universe is curved at a certain curvature'? (Noting that I don't say the centre of the universe is nowhere, I say it is in the past.)

If that's the case, taking into account my note, then I am happy.

If it is also the case that the universe expands in such a way that that expansion can be interpreted as the passage of time, then I am also happy. - Note, I am not saying that the universe is expanding with time, or over time, but effectively that very expansion is time. If that is the generally accepted case, then I am very happy.

Is that the case? If it is then it seems from what you are saying that I have somehow come to this via an unorthodox route, and it involves the idea of what is effectively a hypersurface of simultaneity - one which constitutes the boundary of the universe in terms of four dimensions.

I am sorry that the conversation spins off into weird directions, it is certainly not my intention that it should.

cheers,

neopolitan

PS And as for "embedding", I don't think I used that term at all. It's a bit like reading some of what I typed earlier and writing it off as the "Lorentzian ether interpretation". Giving what I muse about a label doesn't make it what the label says it is, and it doesn't mean that I go along with all the baggage normally associated with the label.
 
  • #163
neopolitan said:
Is it generally agreed that 'the "boundary" of our universe is everywhere and that the "center" of our universe is nowhere and that our universe is curved at a certain curvature'?
It is generally accepted that the center is nowhere (or at least nowhere within the 3D space of the universe), but I'm not sure what the difference is between saying "the boundary of the universe is everywhere" and saying "the universe has no boundary". If we look at a 2D surface without an edge like a flat plane, would you say that the entire surface is its own boundary? Would you say that even if the surface is not embedded in a higher-dimensional space?
neopolitan said:
(Noting that I don't say the centre of the universe is nowhere, I say it is in the past.)
What do you mean "the universe is curved at a certain curvature"? As I said in a previous post, the curvature of 3D space depends on the density of mass and energy--if the density is higher than a certain critical value it has positive spatial curvature, which is analogous to the 2D surface of a sphere, but if it's right at that critical value it'd be flat (zero spatial curvature) like an infinite 2D plane, and if it's below that critical value it'd have negative curvature which is similar to the 2D surface of an infinite saddle. Again, just have a look at the diagrams and explanation here.
neopolitan said:
If it is also the case that the universe expands in such a way that that expansion can be interpreted as the passage of time, then I am also happy. - Note, I am not saying that the universe is expanding with time, or over time, but effectively that very expansion is time. If that is the generally accepted case, then I am very happy.
You should not imagine that time is just the radial dimension of the hypersphere representing a positively-curved space, so that successive moments would be like layers of an onion in a higher-dimensional space, if that's what you're suggesting; this would imply that time has to reverse if the universe begins to contract again (which positively-curved universes naturally do unless the cosmological constant is high enough), but that isn't a prediction of general relativity. If you want to imagine a positively-curved universe that expands from a big bang and then collapses back into a big crunch, it's better to drop the dimensions in your analogy down by one again, so that a hypersurface of simultaneity is represented by a 1D line curved into a circle; then spacetime as a whole would look like the 2D surface of an American football, with one pointy end being the big bang and the other pointy end being the big crunch, and each cross-section of the football surface would give a different-sized circle representing the size of the universe at a particular moment; as you moved from the big bang to the big crunch, the circles would grow bigger for a while, then shrink again. The fact that simultaneity is relative could be represented by the fact that you are free to slice the football at different angles in order to produce your stack of expanding and contracting circles.

Of course this analogy still requires us to imagine the surface embedded in a higher-dimensional space, which should not be taken seriously as anything physical--as I said, the mathematics of differential geometry allows you to describe curvature in purely intrinsic terms without reference to an embedding space, so the notion of space (or spacetime) sitting in some higher-dimensional space becomes physically irrelevant, another "metaphysical" notion like absolute simultaneity (although some variants of superstring theory do imagine the universe as a sort of membrane in a higher dimension, and in this theory the additional dimension does have physical consequences).
neopolitan said:
PS And as for "embedding", I don't think I used that term at all.
But that's the standard term for a higher-dimensional space in which a lower-dimensional curved surface is "sitting", like a curved 3D hypersurface of simultaneity sitting in a 4D (or higher) space. Is this not what you were talking about when you referred to the universe having a "4 dimensional centre" in post #143? The center of a 3D surface which is curved into a hypersphere cannot lie anywhere on the surface itself.
 
Last edited:
  • #164
JesseM said:
It is generally accepted that the center is nowhere (or at least nowhere within the 3D space of the universe), but I'm not sure what the difference is between saying "the boundary of the universe is everywhere" and saying "the universe has no boundary". If we look at a 2D surface without an edge like a flat plane, would you say that the entire surface is its own boundary? Would you say that even if the surface is not embedded in a higher-dimensional space?

No, I said that the surface of a sphere ( a 2D surface ) represents the boundary of the sphere ( a 3D volume ). Only when you think in terms of a 3D hypersurface and a 4D "hypervolume" can you consider that the apparent 3D universe is its own boundary - noting that this phrasing is yours, not mine. I don't disagree with the phrasing, so I am not 100% sure that I didn't use it, but checking back I can't see anywhere where I did.

JesseM said:
What do you mean "the universe is curved at a certain curvature"? As I said in a previous post, the curvature of 3D space depends on the density of mass and energy--if the density is higher than a certain critical value it has positive spatial curvature, which is analogous to the 2D surface of a sphere, but if it's right at that critical value it'd be flat (zero spatial curvature) like an infinite 2D plane, and if it's below that critical value it'd have negative curvature which is similar to the 2D surface of an infinite saddle. Again, just have a look at the diagrams and explanation here.


Actually that phrasing is DaleSpam's.


JesseM said:
You should not imagine that time is just the radial dimension of the hypersphere representing a positively-curved space, so that successive moments would be like layers of an onion in a higher-dimensional space, if that's what you're suggesting; this would imply that time has to reverse if the universe begins to contract again (which positively-curved universes naturally do unless the cosmological constant is high enough) <snip>

I disagree. It does not imply that time has to reverse in order to reach another big-bang event. There is another possibility. I don't have time to go into it right now, but perhaps you can work it out yourself.

Think about the fact that 1) the universe is expanding and 2) the universe is not expanding uniformly. If it were expanding uniformly, we would never notice it because we would expand with it. What is expanding is the space between masses (masses being concentrations of energy).

What happens when the universe is at maximum entropy? I am thinking here about "http://en.wikipedia.org/wiki/Heat_death" [Broken]".

If at this point the universe continues to expand but expands uniformly, and all the energy in the universe is homogenously distributed, it is basically indistinguishable from the entirety of the universe compacted homogenously into a very small volume (in the past). As long as the heat distribution is homogenous nothing will happen, but if the distribution becomes hetrogenous and gap opens up, this gap will expand faster than where the relative concentration of energy is. Maybe this will be overcome and a new equilibrium will be reached, but eventually the gap will open up enough to effectively flip the universe inside out, so that rather than having a small empty space in large otherwise homogenous heat energy distribution, you now have a relatively large empty space surrounding a highly compact concentration of energy. And that then explodes.

Maybe.

I don't know, since I wasn't around when it happened before, and I doubt that I will be here when it happens again. But this does allow me to have my cake and eat it to. The universe continues to expand and I effectively get a heat death and I get a big crunch followed by a big bang.

JesseM said:
But that's the standard term for a higher-dimensional space in which a lower-dimensional curved surface is "sitting", like a curved 3D hypersurface of simultaneity sitting in a 4D (or higher) space. Is this not what you were talking about when you referred to the universe having a "4 dimensional centre" in post #143? The center of a 3D surface which is curved into a hypersphere cannot lie anywhere on the surface itself.

It may be the standard term, but I didn't use it. If I use the standard term, I might get tied to things that I don't intend. Embedding sounds contrived to me. If we can say the two dimensional surface of the Earth is embedded in the apparent 3D universe then I probably can go along with it, but DaleSpam indicated that there is much to study with the concept, so I worry that using the term "embedded" will sign me up for ideas and concepts that I am not aware of.

Yes, the centre of a 3D surface which is curved into a hypersphere cannot lie anywhere on the surface itself.

cheers,

neopolitan
 
Last edited by a moderator:
  • #165
neopolitan said:
No, I said that the surface of a sphere ( a 2D surface ) represents the boundary of the sphere ( a 3D volume ). Only when you think in terms of a 3D hypersurface and a 4D "hypervolume" can you consider that the apparent 3D universe is its own boundary - noting that this phrasing is yours, not mine.
OK, but once again you are assuming that curved 3D space is embedded in a higher-dimensional space, so that we can talk about the "volume" enclosed by 3D space in this higher dimension. This is not an assumption of general relativity--again, general relativity uses differential geometry to describe curved space and curved spacetime without the notion that they are embedded in a higher dimensional space.
neopolitan said:
I disagree. It does not imply that time has to reverse in order to reach another big-bang event.
What does not imply it? Are you indeed imagining that time is just the radial dimension of a hypersphere? If so, then besides the fact that this relies on unphysical notions about embedding space in a higher dimension, and doesn't make sense in the case of a universe with negative or zero curvature (in which case space is not shaped like a hypersphere), I don't see how you could believe this and yet not believe that as the universe shrinks it is returning to earlier times, since the you're now moving towards the center on the radial dimension rather than away from it. But maybe you're not really thinking of time as just the radial dimension in this way, please clarify what you meant by "effectively that very expansion is time".
neopolitan said:
There is another possibility. I don't have time to go into it right now, but perhaps you can work it out yourself.

Think about the fact that 1) the universe is expanding and 2) the universe is not expanding uniformly. If it were expanding uniformly, we would never notice it because we would expand with it. What is expanding is the space between masses (masses being concentrations of energy).
I don't know what you're hinting at, but suffice to say that trying to understand the meaning of time through intuitive embeddings of space is likely to lead only to weird ideas which have nothing to do with the mathematical predictions of general relativity.
neopolitan said:
What happens when the universe is at maximum entropy? I am thinking here about "http://en.wikipedia.org/wiki/Heat_death" [Broken]".

If at this point the universe continues to expand but expands uniformly, and all the energy in the universe is homogenously distributed
That's not what maximum entropy would necessarily look like--for gravitating systems, greater entropy often leads to more clumpiness, not more homogeneity (the amount of clumpiness in the equilibrium distribution will depend on the temperature).
neopolitan said:
As long as the heat distribution is homogenous nothing will happen, but if the distribution becomes hetrogenous and gap opens up, this gap will expand faster than where the relative concentration of energy is.
Why do you assume GR would predict that a gap would expand faster?
neopolitan said:
Maybe this will be overcome and a new equilibrium will be reached, but eventually the gap will open up enough to effectively flip the universe inside out, so that rather than having a small empty space in large otherwise homogenous heat energy distribution, you now have a relatively large empty space surrounding a highly compact concentration of energy. And that then explodes.
What would cause the energy to become "highly compact"? Self-gravity? If so, why would it "explode" rather than just becoming more compact and perhaps forming a black hole?
neopolitan said:
I don't know, since I wasn't around when it happened before, and I doubt that I will be here when it happens again. But this does allow me to have my cake and eat it to. The universe continues to expand and I effectively get a heat death and I get a big crunch followed by a big bang.
Just in a way that follows from your own vague imaginings, not in a way that follows from any well-defined theory of physics, as far as I can tell.
neopolitan said:
It may be the standard term, but I didn't use it. If I use the standard term, I might get tied to things that I don't intend. Embedding sounds contrived to me.
"Contrived" in what way? All that embedding means in this context is having a curved lower-dimensional surface sitting in a noncurved higher-dimensional space or spacetime.
neopolitan said:
If we can say the two dimensional surface of the Earth is embedded in the apparent 3D universe then I probably can go along with it
Yes, of course.
neopolitan said:
but DaleSpam indicated that there is much to study with the concept, so I worry that using the term "embedded" will sign me up for ideas and concepts that I am not aware of.
The "much to study" is just geometry--pure math, not any new physics. For example, it's a nontrivial mathematical result that for any possible curved 4D spacetime (with the curvature defined in terms of differential geometry), it's guaranteed to be embeddable in a flat spacetime with 87 space dimensions and 3 time dimensions.
neopolitan said:
Yes, the centre of a 3D surface which is curved into a hypersphere cannot lie anywhere on the surface itself.
And do you agree it is possible to describe the curvature of a 3D surface with no reference whatsoever to any higher-dimensional space, so the idea that such a higher-dimensional space exists at all is physically superfluous?
 
Last edited by a moderator:
  • #166
neopolitan said:
Is it generally agreed that 'the "boundary" of our universe is everywhere and that the "center" of our universe is nowhere and that our universe is curved at a certain curvature'? (Noting that I don't say the centre of the universe is nowhere, I say it is in the past.)
What I am referring to is the widely accepted "Copernican principle" which basically states that we are probably do not occupy a special point in the universe. My understanding is that, based on what we observe astronomically and on the Copernican principle the general agreement is that at every point in the universe it would look like the universe is expanding in all directions. So there is no center and there is no edge in terms of 3D space. As JesseM mentioned saying that the boundary is everywhere is not much different from saying that there is no boundary.

neopolitan said:
PS And as for "embedding", I don't think I used that term at all. It's a bit like reading some of what I typed earlier and writing it off as the "Lorentzian ether interpretation". Giving what I muse about a label doesn't make it what the label says it is, and it doesn't mean that I go along with all the baggage normally associated with the label.
Objecting to the use of standard terms is pointless. The Lorentz ether is an undetectable but still real absolute reference frame. The concept of a lower-dimensional curved space described from the point of view of a higher-dimensional flat space is embedding. Sorry you don't like the terms, but they are useful for communication, and your ideas are substantially described by those terms.

I don't know why you think using an appropriate label is "writing it off", that is certainly not my intent. I think the Lorentz ether concept is useful for explaining relativistic Doppler effects, and I think embedding is a useful way to understand basic curvature concepts. I see nothing dismissive or otherwise wrong with using the appropriate standard terminology.
 
  • #167
DaleSpam said:
What I am referring to is the widely accepted "Copernican principle" which basically states that we are probably do not occupy a special point in the universe. My understanding is that, based on what we observe astronomically and on the Copernican principle the general agreement is that at every point in the universe it would look like the universe is expanding in all directions. So there is no center and there is no edge in terms of 3D space. As JesseM mentioned saying that the boundary is everywhere is not much different from saying that there is no boundary.

Well, yes and no.

It's the difference between an infinitely large universe with no boundary and a closed universe with no boundary. The Copernican principle could be applied to both, as far as I can work out. I think you might run into problems with an infinitely large universe in that such a universe would then have to have infinite mass in order to avoid there being vantage points from which the mass in the universe seems to be approaching you from one direction and the universe in the other direction is empty.

Are there people who hold to both the Copernical principle and the idea of an open universe? It seems not to work for me.

I agree with JesseM that "saying that the boundary is everywhere is not much different from saying that there is no boundary". If we are happy that it is the same thing from different perspectives there is no problem.

DaleSpam said:
Objecting to the use of standard terms is pointless. The Lorentz ether is an undetectable but still real absolute reference frame. The concept of a lower-dimensional curved space described from the point of view of a higher-dimensional flat space is embedding. Sorry you don't like the terms, but they are useful for communication, and your ideas are substantially described by those terms.

I don't know why you think using an appropriate label is "writing it off", that is certainly not my intent. I think the Lorentz ether concept is useful for explaining relativistic Doppler effects, and I think embedding is a useful way to understand basic curvature concepts. I see nothing dismissive or otherwise wrong with using the appropriate standard terminology.

Perhaps I am too touchy, but I would prefer "what you are describing has much in common with Lorentzian ether theory" rather than "this is essentially Lorentzian ether theory". I am familiar with argumentation techniques which involve a form of guilt by association, and since I have been accused before of looking at things from (too say the least) a slightly different perspective, I don't want to be labelled - especially when there is potential for someone to read some of these posts and go off thinking that I believe in ether (which is usually thought of as the luminiferous stuff that Michelson and Morley were looking for, not something almost entirely conceptual as in the theoretical interpretation that you refer to).

I accept that embedding "substantially describes" my ideas, but just be aware that if you or anyone else make the step "those who espouse embedding also say this", then that step is not valid. Note also that pretty much every single reference to "embedding" that I can find on the internet is linked to brane theory. So, using the term may inadvertently link me to that school of thought - and by extension to M-theory and thence to superstring theory. And as things currently stand, I am sceptical about all sorts of string theory which form a theoretical frame that seems pre-Copernican to me.

cheers,

neopolitan
 
  • #168
JesseM said:
neopolitan said:
JesseM said:
You should not imagine that time is just the radial dimension of the hypersphere representing a positively-curved space, so that successive moments would be like layers of an onion in a higher-dimensional space, if that's what you're suggesting; this would imply that time has to reverse if the universe begins to contract again (which positively-curved universes naturally do unless the cosmological constant is high enough<snip>.

I disagree. It does not imply that time has to reverse in order to reach another big-bang event.

What does not imply it? Are you indeed imagining that time is just the radial dimension of a hypersphere? If so, then besides the fact that this relies on unphysical notions about embedding space in a higher dimension, and doesn't make sense in the case of a universe with negative or zero curvature (in which case space is not shaped like a hypersphere), I don't see how you could believe this and yet not believe that as the universe shrinks it is returning to earlier times, since the you're now moving towards the center on the radial dimension rather than away from it. But maybe you're not really thinking of time as just the radial dimension in this way, please clarify what you meant by "effectively that very expansion is time".

neopolitan said:
If it is also the case that the universe expands in such a way that that expansion can be interpreted as the passage of time, then I am also happy. - Note, I am not saying that the universe is expanding with time, or over time, but effectively that very expansion is time. If that is the generally accepted case, then I am very happy.

Just trying to get this in context, if you pull out bits and ask for explanations out of context, it doesn't really help.

We know that the universe is expanding. We also know that we have this phenomenon which we perceive as the passage of time (we can possibly say that as "we also know that there is this thing called time" or however you prefer to state it).

How do we know that? We can only tell when there is change. Velocities tell us (because things which have unlike velocities experience change in their relative positions).

Imagine for a moment that everything in the universe was stationary (yes, I know it is not possible). Nothing changes and there is, effectively, no passage of time (your clocks don't move, everything is stationary remember).

Now, add in universal expansion. You have time passing again because things are moving apart from each other, there is change. This is sort of what I mean by the very expansion of the universe is time but this is illustrative not prescriptive.

Without considering the whole of what I have said, you won't grasp what I am trying to say. If you think of one moment in time (a "one true simultaneity", one that we cannot distinguish, but which might have some importance), and then the moment after that, you have two moments which you can not only use to observe change, but between which change can occur.

Now if this "when", this one moment in time, is the surface of the universe, then is it subsequently surrounded by the next moment, and is slightly larger. This is universal expansion as passage of time. It will not be observable unless there is something which counteracts or resists this expansion, since even the space between our constituent atoms will expand and our rulers would expand. However, we do notice that the universe doesn't expand uniformly.

JesseM said:
That's (referring to Heat Death) not what maximum entropy would necessarily look like--for gravitating systems, greater entropy often leads to more clumpiness, not more homogeneity (the amount of clumpiness in the equilibrium distribution will depend on the temperature).

Some people seem to think that heat death is where the universe is going. Even some people with far more letters behind their name than I have. Now I am not appealing to authority, but just saying that while I cannot disagree that there are others who think differently, like yourself, I am not alone in thinking that heat death is possible.

JesseM said:
Why do you assume GR would predict that a gap would expand faster?

Which bits of the universe are expanding fastest? Where there is a concentration of mass or where there isn't? I am not using GR to predict this, I am looking at the universe and noting
neopolitan said:
the fact that 1) the universe is expanding and 2) the universe is not expanding uniformly. If it were expanding uniformly, we would never notice it because we would expand with it.

As you can see from the quote, I had already said that.

JesseM said:
What would cause the energy to become "highly compact"? Self-gravity? If so, why would it "explode" rather than just becoming more compact and perhaps forming a black hole?

Well, actually, gravity is possibly a symptom of other factors rather than a cause in itself. In reality the mass of the universe wouldn't be anymore compact than it was previously, but relative to the expanding empty space around it would be highly compact.

As to forming a black hole, there are arguments that the mass of the universe already is a black hole, but on the inside. And anway, your argument here is "why the big bang, why didn't it just turn into a black hole"? It apparently happened once, I can't see why it couldn't happen again.

JesseM said:
"Contrived" in what way? All that embedding means in this context is having a curved lower-dimensional surface sitting in a noncurved higher-dimensional space or spacetime.

It's more a personal thing, embedding sounds active: "General Disorder embedded the journalist Ms Tellall into the troop of soldiers". If it is not meant this way, it is entirely a descriptive term and means something like "the 2D surface of a 3D object is intrinsic to our mathematical description of the object" then I can agree that it isn't contrived.

JesseM said:
And do you agree it is possible to describe the curvature of a 3D surface with no reference whatsoever to any higher-dimensional space, so the idea that such a higher-dimensional space exists at all is physically superfluous?

Not really sure what you are saying here. Is our universe this curved 3D surface? Why do we now ignore 4D space, which is the basis of 4-vector notation which most GR devotees are so fond of? Or are you talking about the surface of a 3D object (which I, perhaps erroneously, refer to as 2D) and want to not mention a fourth dimension? That would be fine by me.

If it is the former, I can't imagine that 4-space is physically superfluous, so sorry, no.

cheers,

neopolitan
 
  • #169
neopolitan said:
Just trying to get this in context, if you pull out bits and ask for explanations out of context, it doesn't really help.

We know that the universe is expanding. We also know that we have this phenomenon which we perceive as the passage of time (we can possibly say that as "we also know that there is this thing called time" or however you prefer to state it).

How do we know that? We can only tell when there is change. Velocities tell us (because things which have unlike velocities experience change in their relative positions).

Imagine for a moment that everything in the universe was stationary (yes, I know it is not possible).
It's possible to have a flat SR spacetime (which is also allowed in GR) where all particles are stationary relative to one another, if that's what you mean.
neopolitan said:
Nothing changes and there is, effectively, no passage of time (your clocks don't move, everything is stationary remember).

Now, add in universal expansion. You have time passing again because things are moving apart from each other, there is change. This is sort of what I mean by the very expansion of the universe is time but this is illustrative not prescriptive.
But do you agree there would equally be change in a non-expanding flat spacetime if particles were simply moving relative to one another? If so, then I still don't get why you would say the expansion "is" time.
neopolitan said:
Without considering the whole of what I have said, you won't grasp what I am trying to say. If you think of one moment in time (a "one true simultaneity", one that we cannot distinguish, but which might have some importance), and then the moment after that, you have two moments which you can not only use to observe change, but between which change can occur.

Now if this "when", this one moment in time, is the surface of the universe, then is it subsequently surrounded by the next moment, and is slightly larger.
But why do you say "surrounded by", if you're not picturing time as the radial axis in a 4D space where the 3D universe at a single instant is embedded? Did you read my analogy where if we picture the universe at any given instant (according to some definition of simultaneity) as a 1D line curved into a circle rather than a 2D surface curved into a sphere, then spacetime as a whole can be pictured as an upright American football, with the bottom point as the big bang and the top point as the big crunch, and each successive cross-section from top to bottom giving a circle that represents the universe at a given instant? In this case, a later moment would lie "above" a previous one in our visualization, it wouldn't surround the previous one like layers of an onion. You can see a fuzzy illustration of this sort of visualization http://www.fortunecity.com/emachines/e11/86/space.html [Broken].
neopolitan said:
This is universal expansion as passage of time.
You still haven't explained why you see expansion as passage of time, rather than just one of the many examples of things in the universe which change over time.
neopolitan said:
It will not be observable unless there is something which counteracts or resists this expansion, since even the space between our constituent atoms will expand and our rulers would expand. However, we do notice that the universe doesn't expand uniformly.
The fact that small bound systems don't expand can be understood in the context of GR (a ruler is held together by non-gravitational forces, but you can also look at gravitationally bound systems like the solar system, which isn't expected to expand with the universe either)--see this section of the Usenet Physics FAQ.
neopolitan said:
JesseM said:
That's (referring to Heat Death) not what maximum entropy would necessarily look like--for gravitating systems, greater entropy often leads to more clumpiness, not more homogeneity (the amount of clumpiness in the equilibrium distribution will depend on the temperature).
Some people seem to think that heat death is where the universe is going. Even some people with far more letters behind their name than I have. Now I am not appealing to authority, but just saying that while I cannot disagree that there are others who think differently, like yourself, I am not alone in thinking that heat death is possible.
You misread what I said there. You inserted the parentheses "(referring to Heat Death)" in my sentence, but I wasn't referring to Heat Death, I was referring to your statement "all the energy in the universe is homogenously distributed". In pure GR, a state of maximum entropy (and 'Heat Death' is the idea that the universe will go to a maximum entropy state, if you didn't know) will not be a homogenous distribution of matter, it will actually be very clumpy, with matter collected into black holes. Of course, if you try to incorporate quantum effects, particularly Hawking radiation which is expected to cause black holes to evaporate into mostly photons, then things get more complicated; in this case, it might again be true that the maximum-entropy state would be pretty homogenous spatially, just a universe filled with photons left over from Hawking radiation (see the photon age from the wikipedia article on heat death).
neopolitan said:
Which bits of the universe are expanding fastest? Where there is a concentration of mass or where there isn't? I am not using GR to predict this, I am looking at the universe and noting
Yes, and as I said, GR can explain this observation. But it's not clear that this is equivalent to the idea that if you have a mostly homogeneous distribution of matter/energy throughout the universe and then a small empty or almost empty region forms in one spot, then this region will begin to expand faster than the rest of the universe (and even if it expands a little faster the difference might not be very significant, I highly doubt that it would be so much faster that 'the gap will open up enough to effectively flip the universe inside out' and force all the matter to occupy a small region as you suggested).
neopolitan said:
As to forming a black hole, there are arguments that the mass of the universe already is a black hole, but on the inside. And anway, your argument here is "why the big bang, why didn't it just turn into a black hole"? It apparently happened once, I can't see why it couldn't happen again.
Look, just making vague speculative arguments based on isolated facts you have read about cosmology is a very bad way to achieve any understanding of physics, I highly discourage this approach as it will tend to lead you into crackpot-land. All cosmological predictions are based on GR, and GR has a perfectly good answer to why concentrating a bunch of matter in one place in a larger space will cause a black hole to form, while the dense but fairly homogenous distribution of matter and energy throughout space in the first moments after the big bang did not form a black hole--read over this section of the Usenet Physics FAQ.
neopolitan said:
It's more a personal thing, embedding sounds active: "General Disorder embedded the journalist Ms Tellall into the troop of soldiers".
That's a pretty recent usage of the word. How about something like "a large emerald was embedded in the king's crown?"
neopolitan said:
If it is not meant this way, it is entirely a descriptive term and means something like "the 2D surface of a 3D object is intrinsic to our mathematical description of the object" then I can agree that it isn't contrived.
The embedding space/spacetime is always assumed to have zero curvature, so regardless of the number of dimensions it's easy to set up a coordinate system with straight orthogonal axes, like a Cartesian coordinate system where all the axes are straight lines that meet at right angles at a single origin. So, the curved surface that is embedded in this embedding space can be completely described in terms of this coordinate system, in the same way that a 2D spherical surface of radius one can be described in a 3D embedding space using the equation x^2 + y^2 + z^2 = 1 (any x,y,z coordinates that lie on the surface of the sphere will satisfy this equation, while x,y,z coordinates that don't lie on the sphere won't satisfy it). That's basically all that embedding space implies, that it's possible to completely describe the curved surface in terms of what points it occupies in a coordinate system in the embedding space.
neopolitan said:
Not really sure what you are saying here. Is our universe this curved 3D surface? Why do we now ignore 4D space, which is the basis of 4-vector notation which most GR devotees are so fond of?
No! There is no 4th dimension of space in GR, only a 4th dimension of time. And the 4th dimension is not an uncurved "embedding space" for curved 3D space; rather, GR describes the curved 4D surface of spacetime in purely intrinsic terms, without the need for a 5th dimension for this curved 4D surface to be embedded in. Just think back to the visualization where I pictured a big bang/big crunch spacetime as the surface of an American football; here I have dropped the number of spatial dimensions by 2, so that spacetime is a curved 2D surface with one space dimension and one time dimension. To visualize this curved 2D surface we intuitively have to picture it sitting in an uncurved (Euclidean) 3D space, but GR could describe its curvature in intrinsic terms, with no need for a higher dimension for the spacetime to be embedded in.
 
Last edited by a moderator:
  • #170
neopolitan said:
I am familiar with argumentation techniques which involve a form of guilt by association, and since I have been accused before of looking at things from (too say the least) a slightly different perspective, I don't want to be labelled - especially when there is potential for someone to read some of these posts and go off thinking that I believe in ether (which is usually thought of as the luminiferous stuff that Michelson and Morley were looking for, not something almost entirely conceptual as in the theoretical interpretation that you refer to).
Fair enough. You are correct, the Lorentz ether theory is championed by some real crackpots that I have encountered on other forums. Unfortunately, they generally are pretty ignorant about its predictions and implications (particularly experimental implications) and therefore tend to apply it incorrectly or just generally spout unrelated nonsense and call it "Lorentz ether". So it is not unreasonable of you to be concerned about guilt by association.

However, there are two differences in this case: 1) generally the crackpots themselves claim to agree with Lorentz in order to lend their idea some authority (which you did not do here) and 2) they misapply it to reach erroneous conclusions (which I did not do here). As mentioned before, I am completely comfortable with people applying and using the Lorentz ether approach as long as they do so correctly. It is not a perjorative in my mind as I use it to understand relativistic Doppler effects. Your idea was essentially the Lorentz ether as correctly applied, not a crackpot bastardization.

neopolitan said:
I accept that embedding "substantially describes" my ideas, but just be aware that if you or anyone else make the step "those who espouse embedding also say this", then that step is not valid.
Also fair enough. Another "guilt by association" argument. If you think I make such a logical fallacy please point it out. In the meantime, my use of the labels makes communication easier and is not intended dismissively.
 
  • #171
Hi DaleSpam
DaleSpam said:
Also fair enough. Another "guilt by association" argument. If you think I make such a logical fallacy please point it out. In the meantime, my use of the labels makes communication easier and is not intended dismissively.

I don't think you were making any logical fallacy yourself, and with the understanding of the risks involved, I accept the use of the labels as aids to communication.

Note the following post, in which I point out that it seems to me that JesseM is misusing your label to associate what I am saying with the assertions of others.

cheers,

neopolitan
 
Last edited:
  • #172
JesseM said:
But do you agree there would equally be change in a non-expanding flat spacetime if particles were simply moving relative to one another? If so, then I still don't get why you would say the expansion "is" time.

Actually I do agree that would equally be change in a non-expanding flat spacetime if particles were simply moving relative to one another, which is why I was careful to say

This is sort of what I mean by the very expansion of the universe is time but this is illustrative not prescriptive.

It's also what I pointed to the need to think of the argument as a whole. Without thinking of the argument as a whole, I don't think it is possible to understand what I am getting at. I am happy to have my argument as a whole shot down, but if someone considers one leg of the table I have made and says "your table won't stand if it only has one leg", I have to agree with them on that specific level even if I know that in a grander scale, my table may well still stand. It's possible that it won't, but the reason won't be that "my table won't stand if it only has one leg".

JesseM said:
But why do you say "surrounded by", if you're not picturing time as the radial axis in a 4D space where the 3D universe at a single instant is embedded?

I do though, with reservations about the term "embedded" as noted in a previous post replying to DaleSpam.

JesseM said:
Did you read my analogy where if we picture the universe at any given instant (according to some definition of simultaneity) as a 1D line curved into a circle rather than a 2D surface curved into a sphere, then spacetime as a whole can be pictured as an upright American football, with the bottom point as the big bang and the top point as the big crunch, and each successive cross-section from top to bottom giving a circle that represents the universe at a given instant? <snip>

Yes I did. It's a piecemeal thing though.

Since I "(picture) time as the radial axis in a 4D space where the 3D universe at a single instant is embedded" then the American Football model doesn't work for me. Additionally, I am not sure that the American Football model produces such a neat explanation for length contraction and it's temporal equivalent (effectively the inverse of time dilation) and the invariance of c that my visualisation does. It may, but I doubt it.

JesseM said:
You misread what I said there. You inserted the parentheses "(referring to Heat Death)" in my sentence, but I wasn't referring to Heat Death, I was referring to your statement "all the energy in the universe is homogenously distributed". In pure GR, a state of maximum entropy (and 'Heat Death' is the idea that the universe will go to a maximum entropy state, if you didn't know) will not be a homogenous distribution of matter, it will actually be very clumpy, with matter collected into black holes. Of course, if you try to incorporate quantum effects, particularly Hawking radiation which is expected to cause black holes to evaporate into mostly photons, then things get more complicated; in this case, it might again be true that the maximum-entropy state would be pretty homogenous spatially, just a universe filled with photons left over from Hawking radiation <snip>

Or all the matter in the universe finally collects into one black hole. Note this is part of the whole, the universe is bounded and unless the black holes are absolutely stationary with respect to each other (ignoring the relative velocity due to expansion) these black holes will eventually collide, if Hawking radiation doesn't make them evapourate. I understand that while Hawking radiation is generally accepted, it is still speculative and has not been inequivocably observed.

All the mass of the universe being in one black hole is not really an issue from my perspective, since I think there is something to the idea that we are already inside the effective event horizon of an extremely supermassive black hole.

I note that you said

JesseM said:
Look, just making vague speculative arguments based on isolated facts you have read about cosmology is a very bad way to achieve any understanding of physics, I highly discourage this approach as it will tend to lead you into crackpot-land.

I guess I should be happy that you make that a prediction, rather than a diagnosis. However, this is not something that I took from isolated readings of cosmology and patched into my visualisation. When I thought about it, I came to the conclusion that what I was thinking was perhaps totally impossible since it implies that we would be inside a black hole. It was after that that I got to hear that someone else had done the maths and that showed that the Schwartzschild radius of the universe's mass matched the universe's radius. The equation is there, the figures are there, you can do the maths yourself. It works.

What you may need to do is explain why the equation doesn't apply in this instance.

JesseM said:
Yes, and as I said, GR can explain this observation. But it's not clear that this is equivalent to the idea that if you have a mostly homogeneous distribution of matter/energy throughout the universe and then a small empty or almost empty region forms in one spot, then this region will begin to expand faster than the rest of the universe (and even if it expands a little faster the difference might not be very significant, I highly doubt that it would be so much faster that 'the gap will open up enough to effectively flip the universe inside out' and force all the matter to occupy a small region as you suggested).

There's no forcing the matter to occupy the small region. It is just that the matter doesn't expand like the (relatively) empty space. Relative to this (relatively) empty space, the matter becomes more and more compact. But it is only a relative thing.

The real issue, one for which I have no explanation is why relative compactness of the energy (which may be in the form of photons) should change its form to the stuff of a big bang. It's possible that your link hints at an answer to that question (only possibly, I am not hinting that does).

JesseM said:
That's a pretty recent usage of the word. How about something like "a large emerald was embedded in the king's crown?"

It's less amusing (no opportunity for a character called General Disorder, for instance), but still equally describing something being done to a pre-existing crown - the crown would still be a crown without the emerald (although, it is possible that without the emerald it is not a king's crown, depending on your definitions and cultural expectations). I can't see that a sphere could exist without its surface. So, I don't think there is an equivalent process by which the surface of the sphere is embedded in the sphere.

JesseM said:
The embedding space/spacetime is always assumed to have zero curvature, so regardless of the number of dimensions it's easy to set up a coordinate system with straight orthogonal axes, like a Cartesian coordinate system where all the axes are straight lines that meet at right angles at a single origin. So, the curved surface that is embedded in this embedding space can be completely described in terms of this coordinate system, in the same way that a 2D spherical surface of radius one can be described in a 3D embedding space using the equation x^2 + y^2 + z^2 = 1 (any x,y,z coordinates that lie on the surface of the sphere will satisfy this equation, while x,y,z coordinates that don't lie on the sphere won't satisfy it). That's basically all that embedding space implies, that it's possible to completely describe the curved surface in terms of what points it occupies in a coordinate system in the embedding space.

I direct your attention to the discussion with DaleSpam, and I direct DaleSpam's attention to this paragraph above. For JesseM, this is relevant:

I accept that embedding "substantially describes" my ideas, but just be aware that if you or anyone else make the step "those who espouse embedding also say this", then that step is not valid.

The coloured section in your paragraph is your equivalent of saying "those who espouse embedding also say this".

JesseM said:
No! There is no 4th dimension of space in GR, only a 4th dimension of time. And the 4th dimension is not an uncurved "embedding space" for curved 3D space; rather, GR describes the curved 4D surface of spacetime in purely intrinsic terms, without the need for a 5th dimension for this curved 4D surface to be embedded in. Just think back to the visualization where I pictured a big bang/big crunch spacetime as the surface of an American football; here I have dropped the number of spatial dimensions by 2, so that spacetime is a curved 2D surface with one space dimension and one time dimension. To visualize this curved 2D surface we intuitively have to picture it sitting in an uncurved (Euclidean) 3D space, but GR could describe its curvature in intrinsic terms, with no need for a higher dimension for the spacetime to be embedded in.

I am pretty sure this is another case of taking the use of the term "embedding" and trying to tie me to something that I don't subscribe to. I should have written 4-space, rather than 4D space. Is that better?

I never mentioned a 5th dimension, except by association when responding to your post where you talked about there being 90 dimensions. I didn't call for another higher dimension for spacetime (3+1) to be embedded in.

I think you have misunderstood me somewhere (or we have misunderstood each other somewhere).

cheers,

neopolitan
 
Last edited:
  • #173
neopolitan said:
It's also what I pointed to the need to think of the argument as a whole. Without thinking of the argument as a whole, I don't think it is possible to understand what I am getting at.
But that's my point, it doesn't seem to me that you've presented any argument as to why we should consider the expansion of space to be time, rather than just one of many things that are changing with time.
neopolitan said:
Since I "(picture) time as the radial axis in a 4D space where the 3D universe at a single instant is embedded" then the American Football model doesn't work for me. Additionally, I am not sure that the American Football model produces such a neat explanation for length contraction and it's temporal equivalent (effectively the inverse of time dilation) and the invariance of c that my visualisation does. It may, but I doubt it.
Just to be clear, are you claiming that your way of thinking about time might just be a good way of visualizing the consequences of general relativity, or are you proposing that general relativity might need to be replaced by a new theory which explains thing differently? If the latter then your speculations don't really belong in this forum.

GR does in fact deal with spacetime as a single curved surface, so it's natural to visualize this by dropping the number of space dimensions by two so we can think about a curved 2D surface like the football (we 3-dimensional creatures can't really visualize a curved 3D surface directly, much less a curved 4D surface).
neopolitan said:
Or all the matter in the universe finally collects into one black hole. Note this is part of the whole, the universe is bounded and unless the black holes are absolutely stationary with respect to each other (ignoring the relative velocity due to expansion) these black holes will eventually collide, if Hawking radiation doesn't make them evapourate. I understand that while Hawking radiation is generally accepted, it is still speculative and has not been inequivocably observed.
True, but then again few of the defining features of black holes that are predicted by GR, like the presence of an event horizon or a singularity, have anything in the way of observational evidence. Very dense, non light-emitting objects have been observed, but without GR there'd be no reason to think they have these features.
neopolitan said:
All the mass of the universe being in one black hole is not really an issue from my perspective, since I think there is something to the idea that we are already inside the effective event horizon of an extremely supermassive black hole.
I'm not aware of any model that says we could be in a black hole, since this would seem incompatible with expansion--the page I linked to suggested the possibility we might be in a giant white hole (the temporal reverse of a black hole, it only spits out matter and nothing can enter its horizon), but even this model would probably be considered fairly far-fetched by most physicists.
neopolitan said:
However, this is not something that I took from isolated readings of cosmology and patched into my visualisation. When I thought about it, I came to the conclusion that what I was thinking was perhaps totally impossible since it implies that we would be inside a black hole. It was after that that I got to hear that someone else had done the maths and that showed that the Schwartzschild radius of the universe's mass matched the universe's radius. The equation is there, the figures are there, you can do the maths yourself. It works.
Unless you have a GR-based model which puts us in a black hole and yet gives the same observational predictions about things like the redshifts of distant galaxies, then this is indeed just spinning ungrounded speculations based on a few isolated facts you have read (namely, your claim that the Schwarzschild radius mass matches the universe's radius--though I'd like to see the source of this, estimates of the radius of the observable universe have changed over the years, and no one claims to know the size of the entire universe beyond the distance that light has had time to get to us, if the curvature of space is flat or negative then mainstream models typically treat it as infinite).
neopolitan said:
What you may need to do is explain why the equation doesn't apply in this instance.
I already pointed out the well-understood fact that in GR, the "Schwarzschild radius" only applies to a non-expanding space, in an expanding universe you can have more mass in a space of that size without a black hole being formed. Did you not read the page I linked to?
JesseM said:
That's a pretty recent usage of the word. How about something like "a large emerald was embedded in the king's crown?"
neopolitan said:
It's less amusing (no opportunity for a character called General Disorder, for instance), but still equally describing something being done to a pre-existing crown - the crown would still be a crown without the emerald (although, it is possible that without the emerald it is not a king's crown, depending on your definitions and cultural expectations). I can't see that a sphere could exist without its surface. So, I don't think there is an equivalent process by which the surface of the sphere is embedded in the sphere.
Who said anything about the surface of the sphere being embedded in the sphere? I was talking about "embedding" the curved 2D surface in 3D euclidean (noncurved) space.
neopolitan said:
I direct your attention to the discussion with DaleSpam, and I direct DaleSpam's attention to this paragraph above. For JesseM, this is relevant:
JesseM said:
The embedding space/spacetime is always assumed to have zero curvature, so regardless of the number of dimensions it's easy to set up a coordinate system with straight orthogonal axes, like a Cartesian coordinate system where all the axes are straight lines that meet at right angles at a single origin. So, the curved surface that is embedded in this embedding space can be completely described in terms of this coordinate system, in the same way that a 2D spherical surface of radius one can be described in a 3D embedding space using the equation x^2 + y^2 + z^2 = 1 (any x,y,z coordinates that lie on the surface of the sphere will satisfy this equation, while x,y,z coordinates that don't lie on the sphere won't satisfy it). That's basically all that embedding space implies, that it's possible to completely describe the curved surface in terms of what points it occupies in a coordinate system in the embedding space.
The coloured section in your paragraph is your equivalent of saying "those who espouse embedding also say this".
So you are imagining the curved 3D surface of space at a particular moment as being part of a 4D spacetime which is also curved? This would not be "embedding", and while I suppose it's true that these 3D surfaces of simultaneity are in some sense "contained in" the curved 4D spacetime (in the same sense that the 1D circles are 'contained in' the 2D football surface), all attempts to make arguments based on how you visualize such things are likely to go badly wrong since we can really only visualize shapes in the context of 3D euclidean geometry. And your arguments about earlier surfaces of simultaneity being "surrounded by" later ones seems to be based on such a concrete Euclidean visualization. Can you say what it means for one 2D surface to "surround" another one if we aren't picturing them in ordinary Euclidean 3D space, but are instead trying to imagine them as being contained in a curved 3D space or spacetime which is impossible for us to visualize directly? Do you think that successive 3D surfaces of simultaneity "surround" each other in a sense that successive 1D circular cross-sections of the curved football surface do not surround each other?
neopolitan said:
I am pretty sure this is another case of taking the use of the term "embedding" and trying to tie me to something that I don't subscribe to.
No, "embedding" can apply either to space or spacetime (all that matters is that the embedding space/spacetime has zero curvature, like Euclidean space or minkowski spacetime), I was just responding to the fact that you seemed to be visualizing the curved surface of space using a geometric picture of a 2D spherical surface sitting in ordinary 3D space.
neopolitan said:
I never mentioned a 5th dimension, except by association when responding to your post where you talked about there being 90 dimensions. I didn't call for another higher dimension for spacetime (3+1) to be embedded in.
I didn't say you had mentioned a 5th dimension--but I had thought you were assuming the higher dimension that curved space was sitting in would itself be uncurved since you seemed to be visualizing it in terms of ideas taken from ordinary Euclidean geometry (like the notion of one surface 'surrounding' another), so a natural extension of this would be that if GR describes curved 4D spacetime, it must be sitting in a higher-dimensional flat space or spacetime. If you never meant to suggest the higher-dimensional space should be uncurved then I misunderstood, but then see my points above about how any attempts at visualizations (which necessarily involve uncurved Euclidean space, since we can't picture curved 3D space directly) should be regarded with suspicion. The specific idea that one surface "surrounds" another doesn't necessarily make sense if the higher-dimensional space/spacetime they are both contained in is itself curved, as in the case of the football where we can't really say which of two circular cross-sections is surrounding the other.
 
Last edited:
  • #174
JesseM said:
But that's my point, it doesn't seem to me that you've presented any argument as to why we should consider the expansion of space to be time, rather than just one of many things that are changing with time.

Because I am heading off to yet another meeting, I can't respond in depth. However, I think it may be worth the couple of minutes it needs to drag this back into perspective.

neopolitan said:
DaleSpam said:
And third, so what? If we were 2D beings living on the surface of a 3D sphere what benefit would we get from projecting our space up into 3 dimensions. We would find that the "boundary" of our universe is everywhere and that the "center" of our universe is nowhere and that our universe is curved at a certain curvature. All of which we could have deduced intrinsically. I don't see the value added by the embedding.
Is it generally agreed that 'the "boundary" of our universe is everywhere and that the "center" of our universe is nowhere and that our universe is curved at a certain curvature'? (Noting that I don't say the centre of the universe is nowhere, I say it is in the past.)

If that's the case, taking into account my note, then I am happy.

If it is also the case that the universe expands in such a way that that expansion can be interpreted as the passage of time, then I am also happy. - Note, I am not saying that the universe is expanding with time, or over time, but effectively that very expansion is time. If that is the generally accepted case, then I am very happy.

Is that the case? If it is then it seems from what you are saying that I have somehow come to this via an unorthodox route, and it involves the idea of what is effectively a hypersurface of simultaneity - one which constitutes the boundary of the universe in terms of four dimensions.

I am sorry that the conversation spins off into weird directions, it is certainly not my intention that it should.

This is where this strand comes from.

I said I would be very happy if the expansion of the universe could be interpreted as the passage of time. I see that, in my visualisation, it works.

As far as I can tell, you haven't actually said that this is not possible. I think you have said that it is not necessary and that it is not something in your (and perhaps the standard) interpretation of GR. So I could make a slight modification to the original and say:

neopolitan (edit) said:
If it were also the case that it is not impossible that the universe could be expanding in such a way that that expansion could be interpreted as the passage of time, then I would be vaguely happy. - Note, I am suggesting the possibility not so much that the universe is expanding with time, or over time, but that the passage of time we experience is a symptom of that very expansion.

I did subsequently try to explain how I visualise things, but there's no real need to convince you. I would much prefer to have you understand and disagree with what I am trying to express than have you agree without understanding. So, there is no should intended from my side (check back to the quote at the beginning, there was a should in there).

cheers,

neopolitan
 
  • #175
neopolitan said:
I said I would be very happy if the expansion of the universe could be interpreted as the passage of time. I see that, in my visualisation, it works.

As far as I can tell, you haven't actually said that this is not possible. I think you have said that it is not necessary and that it is not something in your (and perhaps the standard) interpretation of GR.
Because I still don't understand what you mean when you say the expansion is the passage of time. Put it this way, do you accept that GR allows for the possibility of a non-expanding universe (either static or contracting) where time still passes? If you accept that GR allows for the possibility, is your idea of interpreting the expansion as the passage of time meant to say that GR is wrong that this is a possibility?
 

Similar threads

  • Special and General Relativity
2
Replies
38
Views
2K
  • Special and General Relativity
Replies
20
Views
747
  • Special and General Relativity
2
Replies
51
Views
2K
  • Special and General Relativity
Replies
25
Views
847
  • Special and General Relativity
3
Replies
89
Views
4K
  • Special and General Relativity
Replies
17
Views
440
  • Special and General Relativity
Replies
9
Views
306
  • Special and General Relativity
Replies
12
Views
1K
  • Special and General Relativity
Replies
13
Views
1K
  • Special and General Relativity
Replies
16
Views
615
Back
Top